Vous êtes sur la page 1sur 36

1.

An 18-year-old G1P0 woman is seen in the clinic for a routine prenatal visit at 28 weeks
gestation. She is Caucasian, not of Mediterranean, Asian or Pacific descent. Her prenatal course
has been unremarkable. She has not been taking prenatal vitamins. Her pre-pregnancy weight
was 120 lb. Initial hemoglobin at the first visit at 8 weeks gestation was 12.3 g/dL. Current weight
is 138 lb. No bruising or petechiae are present. After performing a screening complete blood
count (CBC), the results are notable for a white blood count 9,700/mL, hemoglobin 10.6 g/dL,
mean corpuscular volume 88.2 (80.8 – 96.4,) and platelets 215,000/mL. The patient denies
vaginal or rectal bleeding. The best explanation for the patient’s anemia is:

a) Folate deficiency
b) Relative hemodilution of pregnancy
c) Iron deficiency
d) Underlying hemoglobinopathy
e) Chronic medical condition

A 24-year-old G4P2 woman at 34 weeks gestation complains of a cough and whitish sputum for
the last three days. She reports that “everyone in the family has been sick.” Her temperature last
night was 101.5°F. She denies chest pain. She smokes a half-pack of cigarettes per day. She has
a history of asthma with no previous intubations. She uses an albuterol inhaler, although she has
not used it this week. Pertinent physical exam findings: temperature 99.3°F, respiratory rate 14,
pulse 93, blood pressure 113/77, peak expiratory flow rate 430 L/min (baseline documented in
the outpatient chart = 425 L/min) Pharyngeal mucosa is erythematous and injected. Lungs are
clear to auscultation. Pertinent laboratory studies: white blood cell count 8,700; arterial blood
gases on room air (normal ranges in parentheses): pH 7.44 (7.36 – 7.44); PO2 103 mm
Hg(>100), PCO2 26 mm Hg (28 – 32), HCO3 19 mm Hg (22 – 26.) Chest x-ray is normal. What is
the correct interpretation of this arterial blood gas?

a) The woman is hypoventilating


b) This is a compensated respiratory alkalosis
c) This is a compensated metabolic alkalosis.
d) This is an acute metabolic acidosis
e) She should use her albuterol inhaler

A 26-year-old African-American G1P0 woman presents to your office at 7 weeks gestation with
her husband, who is also African-American. The patient’s brother has sickle cell anemia, and has
been hospitalized on numerous occasions with painful crises requiring narcotic pain medication
and blood transfusions. What are the odds that this couple will have a child with sickle cell
anemia, if the carrier rate for sickle cell disease in the African American population is 1/10?

a) 1 in 15
b) 1 in 60
c) 1 in 160
d) 1 in 400
e) 1 in 100

A 30-year-old G1P1 woman with Type 1 diabetes mellitus (White classification, Class B) is at 10
weeks gestation. Her only other significant history is that she still smokes a half a pack of
cigarettes per day. She is concerned that her child may have birth defects. In this patient, the
most likely structural anomaly to be detected by ultrasound on an 18-week detailed fetal anatomic
survey is:

a) Cardiac anomalies
b) Caudal regression malformation
c) Hydrocephalus
d) Microcephaly
e) Limb reductions

A 35-year-old G3P0 woman is at 11 weeks gestation. Her body mass index is 38. She desires to
have chorionic villus sampling (CVS); however, she is worried about losing her pregnancy from
an elective procedure. After a thorough discussion of the risks and benefits of CVS, you inform
the patient that the risk of miscarriage associated with CVS is approximately which of the
following?

a) 0.1%
b) 1%
c) 5%
d) 10%
e) 15%

A 17-year-old female high school student is brought in by her mother for her first gynecologic
exam. She began her menses at age 12 and has had regular periods for the past three years. For
privacy, you ask to examine the patient without her mother. Further history is obtained in the
examination room. She admits that she has been sexually active with her boyfriend for the past
three years. She uses condoms occasionally and is fearful about possible pregnancy. She
requests that her mother not be informed about her sexual activity. On physical exam, she is
anxious, but normally developed. Her pelvic examination reveals no vulvar lesions, minimal non-
malodorous vaginal discharge, and a nulliparous appearing cervix. The bimanual examination
reveals a slightly enlarged uterus, and her adnexa are non-tender and not enlarged. Urine
pregnancy test is negative. Which of the following interventions do you recommend at this point?

a) Pelvic transvaginal ultrasound


b) Obtaining a Pap smear
c) Empiric treatment with doxycycline and ceftriaxone
d) Initiation of oral contraceptives
e) DNA probes for gonorrhea and chlamydia

5. A 22-year-old G0 female college student is scheduled to discuss her recent abnormal Pap
smear which showed atypical squamous cells of undetermined significance (ASCUS.) She has
had negative Pap smears on a yearly basis since age 18. Her only significant gynecologic history
is genital warts that have not responded to treatment with local application of trichloroacetic acid.
She has had eight sexual partners. She uses condoms and oral contraceptives. She has smoked
a pack a day for the past two years. Which of the following is the most appropriate next step in
the management of this patient?

a) HPV typing
b) Repeat Pap smear in 1 year
c) Cone biopsy
d) Cryotherapy
e) Loop Electrosurgical Excision Procedure (LEEP)

7. A 39-year-old female homeless patient presents to the free clinic reporting nontender
plaques on her vulva for about a week. She noticed these plaques when she wiped. No pruritus
or pain is present. She also notes a brownish rash on the palms of her hands. She has lived on
the streets for five years. She admits to IV drug abuse. She was diagnosed as HIV-positive two
years ago, but has not been compliant with suggested treatment. On examination, three elevated
plaques with rolled edges are noted on the vulva. They are nontender. A brown macular rash is
noted on the palms of her hands and the soles of her feet. What is the most appropriate next step
in the management of this patient?

a) Dark-field examination of the vulvar lesions


b) Biopsy of the lesion
c) Colposcopic evaluation of the vulvar lesions
d) Culture the base of the lesion
e) Perform a wet prep

A 38-year-old woman comes to the office because she noted a persistent yellow, frothy discharge
associated with mild external vulvar irritation. She denies any odor. She tried over the counter
anti-fungal medication without success. The discharge has been present for over three months,
gradually increasing in amount. Douching has resulted in temporary relief, but the symptoms
always recur. She has well-controlled hypertension treated with ACE inhibitors. Her partner of two
years has a vasectomy. Pelvic examination reveals mild erythema at the introitus and a copious
yellow frothy discharge fills the vagina. The cervix has erythematous patches on the ectocervix. A
sample of the discharge is examined under the microscope. What is the most likely finding?

a) Strong amine “fishy” odor when KOH applied to sample


b) Marked polymorphonuclear cells with multi-nucleate giant cells
c) Motile ovoid protozoa with flagella
d) Budding yeast and pseudo-hyphae
e) Clue cells

A 23-year-old woman reports having a solitary, painful vulvar lesion that has been present for
three days. This lesion has occurred twice in the past. She states that herpes culture was done
by her doctor during her last exam and was negative. She is getting frustrated in that she does
not know her diagnosis. She has no significant previous medical history. She uses oral
contraceptives and condoms. She has had four sexual partners in her lifetime. On physical
examination, a cluster of three irregular erosions with a superficial crust is noted on the posterior
fourchette. Urine HCG is negative. You suspect recurrent genital herpes. How do you explain the
negative culture?

a) Cultures were taken too early in the course of the lesion


b) The more definitive test would be serum herpes antibody testing
c) The cultures were refrigerated prior to transport to the lab
d) Herpes cultures have a 10-20% false negative rate
e) The herpes virus cannot be recovered with recurrent infections

A 17 year-old woman comes to your office for her first gynecologic visit. She has been sexually
active for the last year and always uses condoms. What is the most appropriate management
regarding Pap smear screening for this patient?

a) Pap smear at age of 21


b) Pap smear at this visit and then anually
c) Pap smear now and every 3 years
d) Pap smear now and then every other year
e) Pap smear at age 18

a
A 40 year-old patient presents for her first annual visit. She denies any new complaints or
symptoms. She has no history of any gynecologic problems. Family history is significant for a
father with hypertension and a mother, deceased, with breast cancer diagnosed at age 36. A
maternal aunt has ovarian cancer. A physical exam is unremarkable. What screening test should
be offered to this patient next?

a) Breast MRI
b) Mammogram
c) Transvaginal pelvic ultrasound
d) Breast ultrasound
e) BRCA-1/BRCA-2 testing

1. A 35-year-old G3P2 woman presents for her initial prenatal care visit at 15 weeks gestation,
according to her last menstrual period. She reports that a home pregnancy test was positive
about 5 weeks ago. Review of her history is unremarkable and her entire family is in good health.
Physical examination reveals a 10-week size uterus. Which of the following is the most
appropriate next step in establishing this pregnancy’s gestational age?

a) Checking fetal heart tones


b) Hysterosonogram
c) Quantitative BHCG
d) Obstetrical ultrasound
e) Quadruple screen

You are scheduled to see a 34-year-old G2P1 woman at 12 weeks gestation. She did not seek
preconception counseling and is worried about delivering a child with Down syndrome, given her
maternal age. She has no significant medical, surgical, family or social history. Which of the
following tests is most effective in screening for Down syndrome in the second trimester?

a) Quadruple screen
b) Triple screen
c) Amniotic fluid for alpha fetoprotein level
d) Maternal serum alpha fetoprotein level
e) Nuchal translucency measurement with serum PAPP-A and free beta-hCG levels

Incorrect!!! The quadruple test (maternal serum alpha fetoprotein, unconjugated estriol,
human chorionic gonadotropin, and inhibin A) is the most effective screening test for Down
syndrome in the second trimester. Down syndrome occurs in about 1 in 800 births in the absence
of prenatal intervention. The efficacy of screening for Down syndrome is improved when
additional components are added to the maternal serum alpha fetoprotein screening. The addition
of unconjugated estriol and human chronic gonadotropin (the Triple screen) results in a 69%
detection rate for Down syndrome. Adding inhibin A to produce a quadruple screen achieves a
detection rate of 80-85%. An amniotic fluid alpha fetoprotein level is unnecessary. Nuchal
translucency measurement with maternal serum PAPP-A and free beta-hCG (known as the
combined test) is a first trimester screen for Down syndrome. It detects approximately 85% of
cases of Down syndrome at a 5% false positive rate.

Use button to the right. ->


Do NOT use browser Forward or Back Buttons!

A 26-year-old G2P1 woman at 26 weeks gestation presents for a routine 50 gram glucose
challenge test. After receiving a one-hour blood glucose value of 144, the patient has a follow up
100 gram 3-hour oral glucose tolerance test with the following plasma values: fasting blood-sugar
102, 1-hour 181, 2-hours 162, and 3-hours 139 (Normal: fasting 95, 1-hour 180, 2-hours 155, 3-
hours 140). Your next step in this patient’s management is:

a) Repeat the glucose tolerance test in early or mid-third trimester


b) Explain to the patient that she has gestational diabetes and begin diet and blood glucose
monitoring
c) Explain to the patient that she has gestational diabetes and begin diet, treatment with oral
hypoglycemic agents and blood glucose monitoring
d) Explain to the patient that she has gestational diabetes and begin diet, treatment with
insulin and blood glucose monitoring
e) Follow the patient as a normal gestation

Incorrect!!! This patient has three values on the 3-hour glucose tolerance test that were
abnormal. The cutoffs for normal are: Fasting blood sugar less than or equal to 95 1-hour less
than or equal to 180 2-hour less than or equal to 155 3-hour less than or equal to 140 Initial
management should include teaching the patient how to monitor her blood glucose levels at
home, on a schedule that would include a fasting blood sugar and 1- or 2-hour post-prandial
values after all three meals, daily. Goals for blood sugar management would be to maintain blood
sugars when fasting below 90 and 1- and 2-hour post-meal values below 120. A repeat glucose
tolerance test would not add any value, as an abnormal test has already been documented. Oral
hypoglycemic agents and insulin are not indicated at this time, as the patient may achieve
adequate glucose levels with diet modification alone. Gestational diabetes varies in prevalence.
The prevalence rate in the United States has varied from 1.4 to 14% in various studies. Risk
factors for high-risk for gestational diabetes include: a previous large baby (greater than 9 lb,) a
history of abnormal glucose tolerance, pre-pregnancy weight of 110% or more of ideal body
weight, and member of an ethnic group with a higher than normal rate of Type 2 diabetes, such
as American Indian or Hispanic descent.

b) Explain to the patient that she has gestational diabetes and begin diet and blood glucose
monitoring would be awarded 10 points

Prepared using: Test Pilot (6.1b43)

Use button to the right. ->


Do NOT use browser Forward or Back Buttons!

A 29-year-old G2P1 woman at 36 weeks gestation is seen in your clinic for management of her
gestational diabetes. Despite diet modification the patient has required insulin to control her
serum glucose levels. She has gained 25 pounds with the pregnancy. She is at risk for all the
following complications, EXCEPT:

a) Polyhydramnios
b) Neonatal hypoglycemia
c) Intrauterine growth restriction
d) Preeclampsia
e) Fetal macrosomia

Incorrect!!! Intrauterine growth restriction is typically seen in women with pre-existing


diabetes and not with gestational diabetes. Shoulder dystocia, metabolic disturbances,
preeclampsia, polyhydramnios and fetal macrosomia are all associated risks of gestational
diabetes.

c) Intrauterine growth restriction would be awarded 10 points

Prepared using: Test Pilot (6.1b43)


Use button to the right. ->
Do NOT use browser Forward or Back Buttons!

A 31-year-old G2P1 woman at 10 weeks gestation presents for prenatal care with an
unplanned pregnancy. This pregnancy was due to a condom accident and the patient was
unaware of the availability of emergency contraception. She receives care from an internist for
hypertension, a seizure disorder and a history of depression and anxiety. She was taking
labetalol, phenobarbital, and citalopram. She is concerned about the teratogenic risks associated
with her medications and medical history. Which of the following statements is true with regard to
this patient�s risk of delivering an infant with a birth defect?

a) She is still at risk for teratogenesis if she continues to take her medications
b) The patient can decrease her risk for teratogenesis if she stops all of the medications that
may be associated with birth defects.
c) She has already passed the period of organogenesis
d) Her risk for teratogenesis is present for the entire pregnancy
e) The patient should not be concerned about the fact that she was taking phenobarbital, as
it is not associated with birth defects

Incorrect!!! The critical period for organogenesis is during the first 8 menstrual weeks. In
order for medications to adversely affect a pregnancy, there must be exposure of the fetus to a
noxious stimulus for a period during a time of active cell differentiation and that the affected tissue
is active differentiation.

c) She has already passed the period of organogenesis would be awarded 10 points

Prepared using: Test Pilot (6.1b43)

A 23-year-old G1P0 woman at 38 weeks gestation, with an uncomplicated pregnancy, presents


to labor and delivery with the complaint of lower abdominal pain and mild nausea for one day.
Fetal kick counts are appropriate. Her review of symptoms is otherwise negative. Vital signs:
temperature 97.8; blood pressure 100/60; pulse: 79; respiratory rate: 13; fetal heart rate: 140s,
reactive, with no decelerations; tocometer: irregular contractions every 2 to 8 minutes; fundal
height: 36 cm; cervix firm, long, closed and posterior. A urine dipstick is completely negative.
Which of the following is the most likely diagnosis in this patient?

a) Appendicitis
b) Intra-amniotic infection
c) Braxton-Hicks contractions
d) Active labor
e) Placental abruption

Correct!!! Braxton Hicks contractions are characterized as short in duration, less intense
than true labor, and the discomfort as being in the lower abdomen and groin areas. A urinary tract
infection is unlikely given the patient’s lack of symptoms and negative urinary dipstick. Preterm
labor is not correct, as the patient is 38 weeks gestation. Active labor is defined by strong, regular
uterine contractions that cause cervical change. This patient’s history does not suggest she is in
active labor. Patients with appendicitis usually present with fever, decreased appetite, nausea
and vomiting. Patients with intra-amniotic infection frequently present with fever, fundal
tenderness, maternal and fetal tachycardia.

Use button to the right. ->


Do NOT use browser Forward or Back Buttons!

16-year-old G1P0 woman at 39 weeks gestation presents to the labor and delivery room
reporting a gush of blood-tinged fluid approximately 5 hours ago and the onset of uterine
contractions shortly thereafter. She says that, over the past hour, the contractions have become
stronger and closer together. After a brief history and physical, the patient is put on the fetal
monitor. The fetal heart rate is 140 to 150 with accelerations and no decelerations. Uterine
contractions are recorded every 2 to 3 minutes. A pelvic exam reveals that the cervix is 4 cm
dilated and 100 percent effaced. Fetal station is 0. After walking around for 30 minutes the patient
is put back in bed after complaining of further discomfort. She requests an epidural. However,
obtaining the fetal heart rate externally has become difficult because the patient cannot lie still.
What is the most appropriate next step in the management of this patient?

a) Placing the epidural


b) Applying a fetal scalp electrode
c) Performing a fetal ultrasound to assess the fetal heart rate
d) Place an intrauterine pressure catheter (IUPC)
e) Recommending a cesarean delivery

Correct!!! If the fetal heart rate cannot be confirmed using external methods, then the most
reliable way to document fetal well-being is to apply a fetal scalp electrode. Putting in an epidural
without confirming fetal status might be dangerous. Although ultrasound will provide information
regarding the fetal heart rate, it is not practical to use this to monitor the fetus continuously while
the epidural is placed. An intrauterine pressure catheter will provide information about the
strength and frequency of the patient’s contractions. It will not provide information regarding the
fetal status. A cesarean section is not indicated at this point. Closer fetal monitoring via a fetal
scalp electrode should be performed.

b) Applying a fetal scalp electrode would be awarded 10 points

Prepared using: Test Pilot (6.1b43)

Use button to the right. ->


Do NOT use browser Forward or Back Buttons!

16-year-old G1P0 woman at 39 weeks gestation presents to the labor and delivery room
reporting a gush of blood-tinged fluid approximately 5 hours ago and the onset of uterine
contractions shortly thereafter. She says that, over the past hour, the contractions have become
stronger and closer together. After a brief history and physical, the patient is put on the fetal
monitor. The fetal heart rate is 140 to 150 with accelerations and no decelerations. Uterine
contractions are recorded every 2 to 3 minutes. A pelvic exam reveals that the cervix is 4 cm
dilated and 100 percent effaced. Fetal station is 0. After walking around for 30 minutes the patient
is put back in bed after complaining of further discomfort. She requests an epidural. However,
obtaining the fetal heart rate externally has become difficult because the patient cannot lie still.
What is the most appropriate next step in the management of this patient?

a) Placing the epidural


b) Applying a fetal scalp electrode
c) Performing a fetal ultrasound to assess the fetal heart rate
d) Place an intrauterine pressure catheter (IUPC)
e) Recommending a cesarean delivery

Correct!!! If the fetal heart rate cannot be confirmed using external methods, then the most
reliable way to document fetal well-being is to apply a fetal scalp electrode. Putting in an epidural
without confirming fetal status might be dangerous. Although ultrasound will provide information
regarding the fetal heart rate, it is not practical to use this to monitor the fetus continuously while
the epidural is placed. An intrauterine pressure catheter will provide information about the
strength and frequency of the patient’s contractions. It will not provide information regarding the
fetal status. A cesarean section is not indicated at this point. Closer fetal monitoring via a fetal
scalp electrode should be performed.
b) Applying a fetal scalp electrode would be awarded 10 points

Prepared using: Test Pilot (6.1b43)

Use button to the right. ->


Do NOT use browser Forward or Back Buttons!

25-year-old G2P1 woman at 38 weeks gestational age presents to labor and delivery with
spontaneous onset of labor and spontaneous rupture of membranes. Cervical exam was 3 cm at
presentation and 5 cm at last check, two hours ago. Presently, patient is very comfortable and
notes mild contractions. You decide to place an intrauterine pressure catheter (IUPC). On
placement, approximately 300 cc of frank blood and amniotic fluid flow out of the vagina. What is
the most appropriate next step in the management of this patient?

a) Emergent cesarean section


b) Withdraw the IUPC, monitor fetus and then replace if tracing reassuring
c) Begin amnioinfusion
d) Begin Pitocin augmentation
e) Keep IUPC in position and connect to tocometer

Incorrect!!! If an intrauterine pressure catheter is placed, and a significant amount of vaginal


bleeding is noted, then it must be assumed that a uterine perforation may have occurred. In this
case, withdrawing the catheter, monitoring the fetus and observing for any signs of fetal
compromise would be the most appropriate management. If the fetal status is found to be
reassuring, then another attempt at placing the catheter may be undertaken.

b) Withdraw the IUPC, monitor fetus and then replace if tracing reassuring would be
awarded 10 points

Prepared using: Test Pilot (6.1b43)

A 30-year-old G2P0 woman with Type I diabetes since age 11 is now delivering at 38 weeks
gestational age. Maternal labs: B+; RPR non-reactive; HBsAg negative; HIV negative; GBS
negative. She has had moderate control of blood sugar during pregnancy. You expect the baby
will be:

a) Large and hypoglycemic


b) Small and hypoglycemic
c) Large and hyperglycemic
d) Small and hyperglycemic
e) Large and require insulin

Incorrect!!! Small babies are more common with Type I diabetes than with gestational
diabetes, and the blood sugar level of all newborns of diabetic mothers should be monitored
closely after delivery, as they are at increased risk for developing hypoglycemia. Macrosomic
(large) infants are typically associated with gestational diabetes.

b) Small and hypoglycemic would be awarded 10 points


While in labor, a 24-year-old G1P0 woman spikes a temperature of 38.6 C and develops fetal
tachycardia. Maternal labs: O+; RPR non-reactive; HBsAg, negative; HIV negative; GBS
unknown. At delivery, you notice a foul smell. What will be the expected appearance of the baby?

a) Vigorous, pink with normal temperature – is this the correct answer?


b) Vigorous, pale with low temperature
c) Lethargic, pink with high temperature
d) Lethargic, pale with low temperature
e) Lethargic, pale with high temperature

Incorrect!!! This patient clearly has chorioamnionitis and the foul smell upon delivery is a
warning sign that the infant can be septic. A septic infant will typically appear pale, lethargic and
have a high temperature.

e) Lethargic, pale with high temperature would be awarded 10 points

A 24-year-old G1P0 woman has just delivered twins, both male, at 37 weeks. On your initial
assessment, you notice the “A” twin is large and plethoric, and the “B” twin is small and pale. A
complete blood count (CBC) is the next most appropriate step because:

a) “A” twin is at high risk for polycythemia


b) “A” twin is at high risk for thrombocytopenia
c) “B” twin is at high risk for thrombocytopenia
d) “B” twin is at high risk for tachycardia
e) “B” twin is at high risk for hyperbilirubinemia

Incorrect!!! This case is suggestive of twin-twin transfusion syndrome. Polycythemia is a


common complication for the plethoric twin. TTTS is a complication of monochorionic
pregnancies. It is characterized by an imbalance in the blood flow through communicating vessels
across a shared placenta leading to underperfusion of the donor twin which becomes anemic and
overperfusion of the recipient which becomes polycythemic. The donor twin often develops IUGR
and oligohydramnios, and the recipient experiences volume overload and polyhydramnios that
may lead to heart failure and hydrops.

a) “A” twin is at high risk for polycythemia would be awarded 10 points

Use button to the right. ->


Do NOT use browser Forward or Back Buttons!

During labor, a 25-year-old G6P2 woman is treated with mepiridine. You do not have a
complete prenatal history but the patient reports the use of marijuana to control nausea during
her pregnancy. She quickly progresses from 4 cm to fully dilated in 1 hour and is now pushing. A
limp unresponsive male infant is delivered. Heart rate is greater than 100 beats per minute. The
infant has no respiratory effort. Which of the following is the most appropriate next step in the
management of this patient?

a) Give positive pressure ventilation and prepare to intubate


b) Give positive pressure ventilation and prepare to give naloxone
c) Give stimulation only and continue to monitor heart rate
d) Suction thoroughly and check heart rate
e) Suction thoroughly and give naloxone

Incorrect!!! You should give positive pressure ventilation and prepare to intubate the child, if
necessary. Poor social history may negate the use of naloxone (Narcan) because the mother
may have a history of narcotic use, and administration of naloxone to the infant can cause life-
threatening withdrawal. Stimulation may not be sufficient for this infant. Suction will not
necessarily stimulate a respiratory effort.

a) Give positive pressure ventilation and prepare to intubate would be awarded 10 points

Prepared using: Test Pilot (6.1b43)

Use button to the right. ->


Do NOT use browser Forward or Back Buttons!

You are at a delivery of a full-term female who, at one minute, has a heart rate greater than
120 beats per minute, is crying, has acrocyanosis, gags when suctioned and is vigorously moving
all four extremities. What is the APGAR score for this infant?

a) 4
b) 5
c) 6
d) 7
e) 9

Incorrect!!! Heart rate= 2, Respiratory rate= 2, Reflex = 2, Activity =2, Color =1. Therefore,
the one-minute APGAR score is 9.

e) 9 would be awarded 10 points

Test Pilot (6.1b43) is Copyright ©2006, The McGraw Hill Companies, All Rights Reserved.

Use button to the right. ->


Do NOT use browser Forward or Back Buttons!

A 21-year-old G1P0 woman delivered a 4000 gram infant by a low-forceps delivery after a
protracted labor course that included a three-hour second stage. Her prenatal course was notable
for development of anemia, poor weight gain and maternal obesity. Following the delivery, the
patient was noted to have a vaginal sulcus laceration and a third-degree perineal laceration,
which required extensive repair. Which of the following factors places this patient at greatest risk
for developing a puerperal infection?

a) Maternal exhaustion
b) Poor nutrition
c) Obesity
d) Iron deficiency
e) Protracted labor

Incorrect!!! Endometritis appearing in a postpartum period is most closely related to the


mode of delivery. Endometritis can be found in less than 3% of vaginal births and this is
contrasted by a 5-10 times higher incidence after Cesarean deliveries. Factors related to
increased rates of infection with a vaginal birth include prolonged labor, prolonged rupture of
membranes, multiple vaginal examinations, internal fetal monitoring, removal of the placenta
manually and low socioeconomic status.

e) Protracted labor would be awarded 10 points

Prepared using: Test Pilot (6.1b43)

Use button to the right. ->


Do NOT use browser Forward or Back Buttons!

A 23-year-old G1P1 woman develops a fever on the third day after an uncomplicated
Cesarean section that was performed secondary to arrest of descent. The only significant finding
on physical exam is moderate breast engorgement and mild uterine fundal tenderness. The most
likely diagnosis in this patient is:

a) Urinary tract infection


b) Mastitis
c) Endometritis
d) Wound cellulitis
e) Septic pelvic thrombophlebitis

Correct!!! The most common cause of postpartum fever is endometritis. The differential
diagnosis includes urinary tract infection, lower genital tract infection, wound infections,
pulmonary infections, thrombophlebitis, and mastitis. Endometritis appearing in a postpartum
period is most closely related to the mode of delivery and occurs after vaginal delivery in
approximately 2 percent of patients and after cesarean delivery in about 10 to 15 percent. Factors
related to increased rates of infection with a vaginal birth include prolonged labor, prolonged
rupture of membranes, multiple vaginal examinations, internal fetal monitoring, removal of the
placenta manually and low socioeconomic status.

c) Endometritis would be awarded 10 points

Prepared using: Test Pilot (6.1b43)

Use button to the right. ->


Do NOT use browser Forward or Back Buttons!

A 35-year-old G4P3 woman comes in for a postpartum visit. She had a normal uncomplicated
vaginal delivery two weeks ago. She has a history of postpartum depression, which required
treatment with antidepressants with her last pregnancy. Which of the following signs or symptoms
of postpartum depression are most useful to distinguish it from postpartum blues and normal
changes that occur after delivery?

a) Anhedonia
b) Crying Spells
c) Ambivalence toward the newborn
d) Sleeplessness
e) Weight loss

Incorrect!!! In addition to the more common symptoms of depression, the postpartum patient
may manifest a sense of incapability of loving her family and manifest ambivalence toward her
infant. Anhedonia is an inability to experience pleasure from normally pleasurable life events such
as eating, exercise, and social or sexual interaction.

c) Ambivalence toward the newborn would be awarded 10 points

Prepared using: Test Pilot (6.1b43)

Use button to the right. ->


Do NOT use browser Forward or Back Buttons!

A 17-year-old G1P1 woman delivered a term infant two days ago. She is not interested in
breastfeeding and she asks for something to suppress lactation. Which of the following is the
safest method of lactation suppression in this patient?

a) Parlodel
b) Breast binding, ice packs and analgesics
c) Medroxyprogesterone acetate
d) Oral contraceptives
e) Express milk by hand until production stops

Incorrect!!! Hormonal interventions for preventing lactation appear to predispose to


thromboembolic events, as well as a significant risk of rebound engorgement. Parlodel
(Bromocriptine), in particular, was associated with hypertension, stroke and seizures. The safest
method still seems to be breast binding, ice packs and analgesics. The patient should avoid
suckling or other means of milk expression, and the natural inhibition of prolactin secretion will
result in breast involution.

b) Breast binding, ice packs and analgesics would be awarded 10 points

Prepared using: Test Pilot (6.1b43)

Use button to the right. ->


Do NOT use browser Forward or Back Buttons!

A 42-year-old G5P4 woman is exclusively breastfeeding her 2 month old baby when she
develops a fever and a red tender wedge-shaped area on the outer quadrant of her left breast.
Which of the following is the most appropriate treatment for this condition?

a) Cessation of breastfeeding for 48 hours


b) Cessation of breastfeeding indefinitely
c) Antibiotics
d) Warm compresses
e) Incision and drainage

Incorrect!!! The patient has a classic picture of mastitis that is usually caused by strep
bacteria from the baby’s mouth. Mastitis is easily treated with antibiotics. The initial choice of
antimicrobial is influenced by the current experience with staphylococcal infections at the
institution. Most are community-acquired organisms, and even staphylococcal infections are
usually sensitive to penicillin or a cephalosporin. If the infection persists, an abscess may ensue
which would require incision and drainage. However, this patient’s presentation is that of simple
mastitis.

c) Antibiotics would be awarded 10 points

Prepared using: Test Pilot (6.1b43)

Use button to the right. ->


Do NOT use browser Forward or Back Buttons!

A 32-year-old G2P2 woman has just had a spontaneous vaginal delivery. She is concerned
that no breast milk is yet being produced when she tries to feed her baby. You reassure her that
colostrum is rich in protein and nutrients, and that her breast milk will come in 2-3 days when
which of the following hormones have been cleared?

a) Estrogen and progesterone


b) Prolactin and oxytocin
c) Human placental lactogen and prolactin
d) Progesterone and prolactin
e) Growth hormone and GnRh

Correct!!! With delivery, there is a rapid and profound decrease in the levels of progesterone
and estrogen, which removes the inhibitory influence of progesterone on the production of alpha-
lactalbumin by the rough endoplasmic reticulum. The increased alpha-lactalbumin serves to
stimulate lactose synthase and ultimately to increase milk lactose. Progesterone withdrawal also
allows prolactin to act unopposed in its stimulation of alpha-lactalbumin production. This may take
up to 2 days.

a) Estrogen and progesterone would be awarded 10 points


Prepared using: Test Pilot (6.1b43)

A 22-year-old G2P1 woman comes to your clinic today with her 3-month-old daughter. She
was breastfeeding without problems until about 2 weeks ago, when she began to experience sore
nipples. The nipples are very sensitive and there is a burning pain in the breasts, which is worse
when feeding. The tips of the nipples are pink and shiny with peeling at the periphery. Which of
the following might best explain this condition?

a) Group A streptrococcal infection


b) Mastitis
c) Herpes infection
d) Too frequent feedings
e) Candidiasis

Incorrect!!! This presentation is classic for candidiasis and should prompt an inspection of
the baby’s oral cavity.

e) Candidiasis would be awarded 10 points

Use button to the right. ->


Do NOT use browser Forward or Back Buttons!

A 33-year-old G5P4 woman just delivered her 4th baby without complications. She had gained
50 pounds during this pregnancy and would like to begin a weight loss program as soon as
possible. She desires long-term effective contraception, because she doubts she wants more
children. She also desires to breastfeed exclusively for 6 months and has had trouble with this in
the past. Which of the following is the most appropriate choice for this patient?

a) Depot medroxyprogesterone
b) Combined estrogen-progestin contraceptives
c) Tubal ligation
d) Intrauterine device (IUD)
e) Essure

Incorrect!!! A paucity of data exists regarding the effect of hormonal contraception on


breastfeeding. There are concerns that hormones, especially estrogen, may have a negative
impact on the quantity or quality of breast milk. Although Depot medroxyprogesterone is
progesterone only contraceptive, it is known to cause weight gain and would not be a good
choice in this patient. The IUD is the best choice because it is long term, but reversible and does
not affect milk production. Tubal ligation and Essure would not be best for a patient who may
desire more children.

d) Intrauterine device (IUD) would be awarded 10 points

Prepared using: Test Pilot (6.1b43)

Use button to the right. ->


Do NOT use browser Forward or Back Buttons!

A 28-year-old G1P1 woman delivered 3 days ago and desires to breastfeed her infant, but is
having problems since her milk came in with full tender breasts. She calls on the phone and you
tell her that this is normal and is called engorgement. Which of the following strategies may help
relieve her discomfort?

a) Discontinue breastfeeding for 24 hours to decrease the milk supply


b) Cover the breast with cool lettuce leaves
c) Increase the interval between breast feeding sessions to decrease the milk supply
d) Nurse every 1.5-3 hours around the clock
e) Don’t wear a bra until the engorgement subsides

Incorrect!!! Engorgement commonly occurs when milk comes in. Strategies that may help
include frequent nursing, taking a warm shower or warm compresses to enhance milk flow,
massaging the breast and hand expressing some milk to soften the breast, wearing a good
support bra and using an analgesic 20 minutes before breast feeding.

d) Nurse every 1.5-3 hours around the clock would be awarded 10 points

Prepared using: Test Pilot (6.1b43)

Use button to the right. ->


Do NOT use browser Forward or Back Buttons!

A 30-year-old G3P2 woman had her last normal menstrual period 8 weeks ago. She began
spotting 3 days ago and developed cramping this morning. She has a history of a chlamydia
infection with a previous pregnancy. She has no history of other sexually transmitted infections
and is currently sexually active with one partner. She smokes one pack of cigarettes per day and
denies alcohol or drug use. On physical exam, her blood pressure is 120/70, pulse 82,
respirations 20, and temperature 98.5°F. On abdominal exam, she has suprapubic and diffuse
bilateral lower quadrant tenderness, but no rebound or guarding. On pelvic exam, she has normal
external genitalia, old blood in the vaginal vault, the cervix is without lesions and os closed.
Pertinent labs: Quantitative beta hCG = 1000mIU/ml; urinalysis normal; hematocrit = 32%;
transvaginal ultrasound shows no intrauterine pregnancy, no adnexal masses, no free fluid in
pelvis. Which of the following is the most appropriate next step in the management of this
patient?

a) Treat with Methotrexate


b) Exploratory surgery
c) Repeat hCG in 48 hours
d) Repeat hCG in one week
e) Admit the patient to the hospital for observation

Correct!!! The patient first needs to have an accurate diagnosis before a treatment plan is
entertained. She does not yet have a diagnosis of ectopic pregnancy. Repeating the hCG is the
next step in this patient’s management. Inappropriately rising hCG levels (less than 50% increase
in 48 hours) or levels that either do not fall following diagnostic dilatation and curettage (D&C)
would be consistent with the diagnosis of ectopic pregnancy. Alternatively, a fetal pole must be
visualized outside the uterus on ultrasound. The patient would need an hCG level over the
discriminatory zone (the level where an intrauterine pregnancy can be seen on ultrasound) with
an empty uterus. The level commonly used is 2000mIU/ml). Treatment with methotrexate may be
appropriate, but only after a definitive diagnosis is made. The patient does not yet have this level
and is stable. She is, therefore, not a candidate for exploratory surgery. If she had unstable vital
signs or an acute abdomen, a diagnostic laparoscopy/laparotomy would be indicated. Diagnostic
D&C can be done in the instance where the patient has inappropriately rising hCG levels.
Repeating the ultrasound in one week is not recommended because a delay in diagnosis could
result in a ruptured ectopic pregnancy and increased risk to the patient. The patient is stable;
therefore, she does not need to be admitted to the hospital.

c) Repeat hCG in 48 hours would be awarded 10 points

Prepared using: Test Pilot (6.1b43)


A 20-year-old G1P0 woman has vaginal spotting and mild cramping for the last 3 days. She had
her last normal menstrual period approximately 6-1/2 weeks ago. She had a positive home
pregnancy test. Her medical and gynecologic histories are negative and non-contributory. On
physical exam: blood pressure 120/72; pulse 64; respirations 18; temperature 98.0°F. On pelvic
exam, she has scant old blood in the vagina, with a normal appearing cervix and no discharge.
On bimanual exam, her uterus is nontender and small, and there are no adnexal masses
palpable. Pertinent labs: Quantitative BHCG=750 mIU/ml 48 hours ago; today, the level is 760
mIU/ml; progesterone level = 3.2 ng/ml; hematocrit is 37%. Transvaginal ultrasound shows a fluid
collection in the uterus and no fetal pole, no masses and no free fluid in the pelvis. Which of the
following is the most appropriate next step in the management of this patient?

a) Exploratory surgery
b) Treat with Methotrexate
c) Treat with Mifepristone
d) Dilatation and Curettage
e) Repeat hCG level in one week

A 23-year-old G1P0 woman has cramping, vaginal bleeding and right lower quadrant pain. Her
last normal menstrual period occurred 7 weeks ago. On physical exam, vital signs are: blood
pressure 110/74; pulse 82; respirations 18; and temperature 98.8°F. On abdominal exam, she
has very mild right lower quadrant tenderness. On pelvic exam, she has scant old blood in the
vagina and a normal appearing cervix. Her uterus is normal size and slightly tender. On bimanual
exam, there is no cervical motion tenderness, and she has slight tenderness in right lower
quadrant. Pertinent labs: Quantitative beta hCG=2500 mIU/ml; progesterone level = 6.2 ng/ml;
hematocrit = 34%. The transvaginal ultrasound shows an empty uterus with endometrial
thickening, a mass in the right ovary measuring 3.0 x 2 cm and a small amount of free fluid in the
pelvis. Which of the following is the most appropriate next step in the management of this
patient?

a) Methotrexate
b) Antibiotics, repeat the hCG level in 48 hours
c) Observation, repeat the ultrasound in 48 hours
d) Dilation and curettage
e) Culdocentesis
a

A 19-year-old G1P0 woman with a desired pregnancy notes vaginal spotting early this morning
and it has slightly increased. Her last normal menstrual period occurred 6 weeks ago. She has no
pain or other symptoms. Her medical history is noncontributory. On physical exam: blood
pressure is 120/68; pulse 68; respirations 20; and temperature 98.6°F. On pelvic exam, her cervix
is normal; her uterus is small and nontender; there are no masses palpable. Labs show
quantitative BhCG 750 mIU/ml; progesterone level 3.8 ng/ml; hematocrit is 38%. Which of the
following is the most appropriate next step in the management of this patient?

a) Order a transvaginal ultrasound


b) Repeat BhCG level in 24 hours
c) Repeat BhCG level in 48 hours
d) Dilation and curettage
e) Recommend that the patient remain on bed and pelvic rest of the next 24 hours

10. A 19-year-old G1P0 woman notes vaginal spotting. Her last normal menstrual period occurred
6 weeks ago. She began having spotting early this morning and it has increased only slightly. She
has no pain and denies other symptoms. Her medical history is noncontributory. On physical
exam: blood pressure is 120/68; pulse 68; respirations 20; and temperature 98.6°F. On pelvic
exam, her cervix is normal; uterus is small and nontender; and no masses are palpable. Initial
labs show quantitative beta hCG 2000 mIU/ml and hematocrit is 38%. A repeat hCG level 48
hours later is 2100 mIU/ml. A transvaginal ultrasound shows an empty uterus with a thin
endometrial stripe and no adnexal masses. What is your next step in management?

a) Dilatation and Curettage


b) Treat with Methotrexate
c) Exploratory laparotomy
d) Repeat hCG level in 48 hours
e) Repeat ultrasound in 24 hours

ectopic- b

A 27-year-old G2P0 woman is diagnosed with an early first trimester spontaneous abortion. She
has a history of Type I Diabetes Mellitus, mild chronic hypertension and one prior termination of
pregnancy. The patient is very upset because she thinks she will never have a baby and wants to
know what caused this to happen. Which of the following is the most likely cause of this
spontaneous abortion?

a) Prior termination of pregnancy


b) Chronic hypertension
c) Diabetes mellitus
d) Intrauterine adhesions
e) Infection
c

A 22-year-old G1P0 woman presents to the emergency department at eight weeks gestation
experiencing heavy vaginal bleeding. Pelvic exam demonstrates brisk bleeding through a dilated
cervical os. The patient’s hemoglobin is 7 (hematocrit 21%.) Which of the following is the most
appropriate next step in the management of this patient?

a) Administration of intravaginal Misoprostol


b) Administration of oral Misoprostol
c) Dilatation and suction curettage
d) Endometrial ablation
e) Expectant care to permit spontaneous abortion

A 34-year-old female patient originally presented with slight spotting. A sonogram and serial
quantitative BhCG tests have confirmed a 6-week missed abortion. She has no other complaints
or relevant gynecologic, medical or surgical histories. Her partner accompanies her and is
supportive. The patient wishes to avoid any unnecessary medical interventions and asks whether
she can safely let nature take its course. What is the best advice for this patient?

a) She should undergo immediate dilation and suction curettage to prevent severe hemorrhage
b) She should undergo treatment in one week to prevent infection
c) She should undergo immediate medical treatment to lessen surgical risk
d) The patient does not require any treatment over the next couple of weeks
e) The patient should undergo a dilatation and evacuation in one week

A 28-year-old G0 woman presents to your office for preconception counseling. She has a history
of Type I diabetes, diagnosed at age 6, and uses an insulin pump for glycemic control. She has a
history of proliferative retinopathy treated with laser. Her last ophthalmologic examination was 3
months ago. Her last hemoglobin A1C (glycosylated hemoglobin level) 6 months ago was 9.2%.
Which of the following complications is of most concern for her planned pregnancy?

a) Oligohydramnios
b) Fetal growth restriction
c) Fetal cardiac arrhythmia
d) Group B Streptococcal infection
e) Post term pregnancy

A 19-year-old G1 woman at 36 weeks gestation presents for her first prenatal visit, stating she
was recently diagnosed with HIV. She was tested because her former partner recently tested
positive as well. The HIV Western Blot is positive. The CD4 count is 612 cells/µl. The viral load is
9,873 viral particles per ml of patient serum. Which of the management options would best
decrease the risk for perinatal transmission of HIV?

a) Treatment with intravenous Zidovudine at the time of delivery


b) Treatment of the newborn with oral Zidovudine only if HIV-positive
c) One week maternal treatment with Zidovudine now
d) Cesarean section in second stage of labor
e) Single drug therapy to minimize drug resistance

A 19-year-old G1P0 woman at 18 weeks gestation presents with a 3-month history of palpitations
and intermittent chest pain. Physical examination reveals a pulse of 96 and grade II/VI systolic
ejection murmur with a click. The ECG shows normal rate and rhythm and an echocardiogram is
ordered. Which of the following is the best treatment in the management of this patient?

a) anxiolytics
b) β-blockers
c) calcium-channel blockers
d) digitalis
e) no treatment needed at this time

An 18-year-old nulliparous African-American woman was admitted 4 days ago because of back
pain, chills and fever. She has been taking broad spectrum antibiotics but continues to have
spiking fever ranging from 38.9 to 39.6°C. Work-up reveals a right ureteral obstruction secondary
to calculi. Which of the following is the most appropriate next step in the management of this
patient?

a) aggressive hydration
b) change antibiotics
c) continue present antibiotics
d) pass a double-J ureteral stent
e) perform percutaneous nephrostomy

An 18-year-old nulliparous woman presents for her 32 week visit. She complains of intense
itching for the past 2 weeks and cannot stop scratching her arms, legs, and soles of her feet. She
has tried over the counter lotions with no relief. She also states that her family noticed she is
slightly yellow. Her vital signs are normal and there are general excoriations over her arms and
legs. Which of the following is the best treatment in the management of this patient?

a) Aggressive hydration
b) Antivirals such as Acyclovir
c) Antihistamines
d) Cholestyramine
e) Ursodeoxycholic acid

A 36-year-old G5P4 woman with no prenatal care presented in active labor with a blood
pressure of 170/105 and 3+ proteinuria. Fetal heart tones were found to be in the 170s with
decreased variability and a sinusoidal pattern. Resting uterine tone was noted to be increased
and she was having frequent contractions (every 1-2 minutes.) The patient complained of bright
red vaginal bleeding for the past hour. Based on this history, what is the etiology of her vaginal
bleeding?

a) Uterine rupture
b) Placenta previa
c) Bloody show
d) Abruptio placenta
e) Cervical trauma

Incorrect!!! Although all the options above can result in third trimester vaginal bleeding, the
most likely cause in this patient is placental abruption. This diagnosis goes along with the
tachysystole on tocometer and evidence of fetal anemia (tachycardia and sinusoidal heart rate
pattern) on the heart rate tracing. Hypertension and preeclampsia are risk factors for abruption.
She has no history of cervical trauma.

A 24-year-old G2P1 Rh-negative female patient is found at 10 weeks gestation to have anti-D
antibodies. You follow her closely during this pregnancy and order serial ultrasound
examinations. Which of the following fetal ultrasound findings would be most explained by the
presence of Rh disease?

a) Meconium
b) Fetal bladder obstruction
c) Oligohydramnios
d) Pericardial effusion
e) Placenta previa

Incorrect!!! Fetal hydrops is easily diagnosed on ultrasound. It develops in the presence of


decreased hepatic protein production. It is defined as a collection of fluid in two or more body
cavities, such as ascites, pericardial and/or pleural fluid and scalp edema. On occasion, when
extramedullary hematopoiesis is extensive, there will be evidence of hepatosplenomegaly.
Placentomegaly (placental edema) and polyhydramnios are also seen on ultrasound. Meconium,
fetal bladder obstruction, oligohydramnios and placenta previa do not fit the clinical scenario.

A 25-year-old G2P1 woman states her gestational age by a first trimester ultrasound is 16 weeks,
3 days. She reports no complaints and is not yet feeling fetal movement. Her fundal height is just
above the umbilicus and measures 22 cm. The MSAFP (maternal serum alpha fetoprotein) result
is elevated. Which of the following is the most likely cause for the abnormal MSAFP result?

a) Fetal trisomy
b) Dates in error
c) Twin gestation
d) Fetal gastroschisis
e) Fetal demise

A 23 year-old G1P1 delivered vaginally a 42-week infant after a prolonged induction of labor. She
had an epidural, with an indwelling catheter for 36 hours and three IV sites for her intravenous
medications. She now complains of lower abdominal pain, frequency and dysuria. Her vital signs
are temperature 98.6°F , 37°C; pulse 70; blood pressure 100/60; and respirations 12. On
examination, her lungs are clear, cardiac exam is normal, abdomen is soft, uterine fundus is firm
and nontender, and she has mild suprapubic tenderness. Which of the following organisms is
most likely causing her discomfort?

a) Group A streptococcus
b) Gardnerella vaginalis
c) Chlamydia trachomatis
d) Escherichia coli
e) Group B Streptococcus

A 23 year-old G1P1 is 5 days post-operative from a Cesarean section for arrest of labor at 7
centimeters. She now complains of minimal abdominal pain and drainage from the right side of
the incision. Lochia is normal and she has no urinary complaints. Her vital signs are normal and
she is afebrile. On physical exam, her lung and cardiac examinations are normal. Her abdomen
and uterine fundus are nontender. Her Pfannenstiel incision has erythema extending 3
centimeters from the incision and there is purulent, bloody drainage coming from the right side.
What is the next best step in the management of this patient?

a) Initiate intravenous antibiotics


b) Initiate oral antibiotics
c) Occlusive dressing to the wound
d) Open drainage of wound
e) Tropical antibiotics to the wound

22 year-old delivered her first baby five days ago after a prolonged labor and subsequent
Cesarean section for arrest of cervical dilation at 7 centimeters. Fever was noted on
postoperative day 2 and, despite broad spectrum antibiotics, she continues to have temperature
spikes above 101.3¢XF, 38.5„aC. She is eating a normal diet and ambulating normally. On
physical examination, her breasts have no erythema and nipples are intact. Her abdomen is soft,
uterine fundus is firm and nontender, and her incision is healing without induration or erythema.
She has normal lochia and her urinalysis is normal. Pelvic examination reveals a firm nontender
uterus and no adnexal masses or tenderness. Which of the following is the most likely cause of
her fevers?

a) Endometritis
b) Cystitis
c) Septic pelvic thrombophlebitis
d) Ovarian abscess
e) Mastitis

Incorrect!!! Septic thrombophlebitis involves thrombosis of the venous system of the pelvis.
Diagnosis is often one of exclusion of other causes, but sometimes a CT scan will reveal
thrombosed veins. Treatment requires addition of anticoagulation to antibiotics and resolution of
fevers is rapid. Anticoagulation treatment is short-term. Classic clinical findings for endometritis
include fever and maternal tachycardia, uterine tenderness and no other localizing signs of
infection. The clinical manifestations of cystitis include lower abdominal pain, frequency, urgency
and dysuria. The clinical findings in patients with mastitis include fever, tenderness, induration
and erythema of the affected breast.

c) Septic pelvic thrombophlebitis would be awarded 10 points

A 19-year-old G1P0 woman at 40 weeks gestational age has an uncomplicated vaginal


delivery followed by a brisk hemorrhage. Her past medical history is significant for steroid-
dependent asthma. Her blood pressure is 110/70; pulse 84; and she is afebrile. Which of the
following uterotonic agents should not be used in this patient?

a) Intramuscular Oxytocin
b) Methylergonovine
c) Prostaglandin F2
d) Intravenous Oxytocin
e) Misoprostol

Correct!!! Methergine, Prostaglandins and Oxytocin are all uterotonics and used to increase
uterine contractions and decrease uterine bleeding. Prostaglandin F2 (Hemabate) is a potent
smooth muscle constrictor, which also has bronchio-constrictive effect. As such, it should be used
with caution in any patient with reported history of asthma. It is absolutely contra-indicated in
patients with poorly controlled or severe asthma. Misoprostol, non-FDA approved, is used for
cervical ripening and labor induction.

4. A 29-year-old G2 P1 woman presents in early labor after spontaneous rupture of the fetal
membranes. Thirty minutes after arrival, she delivers a 2650 gram male infant. A globular pale
mass appears at the introitus when attempting to deliver the placenta. Her blood pressure is
90/60; pulse 104; and temperature is 37°C. What is the most likely etiology for this event in this
patient?

a) Multiparity
b) Twin gestation
c) Leiomyoma
d) Uterine inversion
e) Rapid labor

8. A 28-year-old G2P1 woman presents in labor at term and delivers a 3500gm male
spontaneously after Oxytocin augmentation of labor. Thirty minutes later, the placenta has not
delivered. Her past medical history is significant for leiomyoma uteri and male factor infertility. Her
prenatal course was uncomplicated. What is the most likely risk factor for retained placenta in this
case?

a) Placenta abruption
b) Labor augmentation
c) Leiomyomas
d) Multiparity
e) Circumvallate placenta

A 37-year-old G4P3 woman presents in labor at term. Her prenatal course was uncomplicated.
She delivers a 3500gm male spontaneously after Oxytocin augmentation of labor. Immediately
postpartum, there is excessive bleeding greater than 2000cc. There are no lacerations and the
uterus is found to be floppy. Which of the following is the most appropriate next step in the
management of this patient?

a) Intramuscular methylergonovine
b) Intravenous methylergonovine
c) Intravenous oytocin bolus
d) Intramuscular prostaglandin F2
e) Intravenous prostaglandin F2

Correct!!! Prostaglandin F2 should be administered intramuscularly. It could also be injected


directly into the uterine muscle. Neither prostaglandin F2 nor methylergonovine should ever be
administered IV, as they can lead to severe bronchoconstriction and stroke, respectively.
Oxytocin is administered as a rapid infusion of a dilute solution (20-80 units in a liter) and not as
an IV bolus.

d) Intramuscular prostaglandin F2 would be awarded 10 points

A 29-year-old G1P0 woman at 31 weeks gestation presents with watery discharge from the
vagina commencing several hours ago. Her prenatal course has been uncomplicated and she
takes prenatal vitamins and iron. She denies substance abuse, smoking or alcohol use. Her blood
pressure is 110/70; pulse 84; temperature 98°F. Which of the following is the most appropriate
next step in the management of this patient?

a) Nitrazine testing of mucus swabbed from cervix


b) Microscopic examination of smear of vaginal fluid to evaluate for ferning
c) Digital examination of cervix to determine if palpable bag of water is present ahead of
presenting part
d) Ultrasound examination to determine if oligohydramnios is present
e) Non-stress test to look for variable decelerations

Incorrect!!! Methods to confirm rupture of membranes include testing the vaginal fluid for
ferning and Nitrazine testing. It is important to test the fluid from the vagina and not to test
cervical mucus because of false positive ferning patterns. While a palpable bag of water would go
against rupture (unless there was a high leak) a digital exam should be avoided in a patient you
suspect might have preterm rupture of membranes because of the risk of introducing bacteria into
the uterine cavity and, thus, chorioamnionitis. Ultrasound may reveal oligohydramnios and
support the diagnosis of rupture of membranes, but does not confirm this diagnosis. Similarly,
non-stress test may reveal variable decelerations, which may be present in the setting of rupture
of membranes.

A 19-year-old G2P1 woman at 30 weeks gestation presents with preterm rupture of membranes 6
hours ago. Her prenatal course has been complicated by two episodes of bacterial vaginosis for
which she was treated. She takes prenatal vitamins and iron. She denies substance abuse or
alcohol use, but admits to smoking five cigarettes each day. Her prior pregnancy was delivered
vaginally at 41 weeks after spontaneous rupture of membranes. Her blood pressure is 110/70;
pulse 84; temperature 98°F. Pertinent sonographic findings reveal oligohydramnios and a cervical
length of 34mm. Which of the following is the most likely cause of preterm premature rupture of
membranes in this patient?

a) Genital tract infections


b) Short cervical length
c) Presence of oligohydramnios
d) Smoking
e) Previous preterm rupture of membranes

A 33-year-old G2P1 woman at 29 weeks gestation presents with confirmed preterm rupture of
membranes. She denies labor. She takes prenatal vitamins and iron. She denies substance
abuse, smoking or alcohol use. Her prior pregnancy was delivered vaginally at 41 weeks after
spontaneous rupture of membranes. Her blood pressure is 110/70; pulse 84; temperature 99°F.
Which of the following will prolong the latency period by approximately 7 days?

a) Antibiotics
b) Corticosteroids
c) Calcium channel blocker
d) Beta mimetics
e) Magnesium sulfate

Incorrect!!! Antibiotic therapy given to patients with preterm premature rupture of the
membranes has been found to prolong the latency by up to 5-7 days, as well as reduce the
incidence of maternal amnionitis and neonatal sepsis. Corticosteroids and tocolytics may also
prolong the pregnancy for various lengths of time, but generally not 7 days.

A 28-year-old primigravid woman at 33 weeks gestation presents with complaints of fluid leaking
from the vagina. Preterm premature rupture of the membrane is diagnosed. The patient has mild
uterine tenderness and you suspect early chorioamnionitis. An amniocentesis is performed.
Which of the following amniotic fluid results is indicative of intra-amniotic infection?

a) Presence of leucocytes
b) Low Interleukin –6
c) Amniotic glucose less than 20 mg/dl
d) Presence of phosphatidylglycerol
e) Low Interleukin-12

1. A 19-year-old G1 woman at 28 weeks gestation comes to Labor and Delivery because of the
onset of contractions. The patient describes the contractions as progressively becoming more
painful, each lasting 40 seconds and now occurring every 5 minutes. She reports good fetal
movement and does not have any bleeding or leakage of fluid. On evaluation in triage, it is noted
that she is having regular contractions, approximately every 5 minutes, has intact membranes
and her cervical exam is 3 cm dilated and 50% effaced. What is the most frequent cause of this
condition?

a) Dehydration
b) Rigorous fetal movement
c) Idiopathic
d) Uterine Distortion
e) Iatrogenic

A 20-year-old G2P1 woman at 32 weeks gestation presents to Labor and Delivery with
contractions every 4 minutes. On physical examination, her vital signs are: temperature 38.1;
heart rate 120; respiratory rate 18, and blood pressure 110/65. Her fundus is non-tender and the
rest of the physical exam is benign. Her cervix is dilated 2 cm and is 50% effaced. Fetal heart
tones are in the 140s and are reassuring, overall. Her white blood cell count (WBC) is 18,000.
Which of the following is the most appropriate next step in the management of this patient?

a) Continue close observation


b) Perform amniocentesis
c) Treat with steroids
d) Begin labor induction
e) Perform an urgent c-section

Incorrect!!! An amniocentesis should be performed to rule out an intra-amniotic infection.


This patient has an unexplained fever and elevated white blood cell count, which are concerning
for an intra-amniotic infection. Delivery is not warranted at this time, as it is possible that the fever
and elevated white blood cell count are from another source. Steroids should not be used until an
intra-amniotic infection has been ruled out. Conservative management with observation would
delay diagnosis and would not be appropriate.

b) Perform amniocentesis would be awarded 10 points

A 26-year-old G2P1 woman presents in preterm labor at 33 weeks gestation. She has a history
of a prior preterm birth at 32-weeks gestation. She has insulin dependent diabetes and has a
history of myasthenia gravis. She has regular contractions every 3 minutes and fetal heart tones
are reassuring. Cervix is 3cm dilated and 0 station. Her blood pressure is 140/90, which of the
following is the most appropriate tocolytic to use in this patient?

a) Calcium channel blocker


b) Terbutaline
c) Magnesium sulfate
d) Indomethacin
e) Ritodrine

Incorrect!!! Calcium channel blocker is the best option for her as she has contraindications to
the other agents listed. Terbutaline and ritodrine are contraindicated in diabetic patients,
magnesium sulfate is contraindicated in myasthenia gravis, and indomethacin is contraindicated
at 33 weeks due to risk of premature ductus arteriosus closure.

a) Calcium channel blocker would be awarded 10 points

Use button to the right. ->


Do NOT use browser Forward or Back Buttons!

A 19-year-old G1 woman presents to the hospital. She states she is 25-weeks pregnant and
complains of vaginal bleeding for the past hour. She had intercourse earlier without discomfort.
Currently she denies cramping or pain and feels the baby moving. Her prenatal course has been
uncomplicated. She takes no medication besides prenatal vitamins and denies smoking, alcohol
or drug use. Her blood pressure is 110/70, pulse 68, and she is afebrile. Her abdomen and uterus
are soft and nontender. Fetal heart tones are in the 150s. Which of the following is the most
appropriate next step in the management of this patient?
a) Vaginal examination
b) Urgent Cesarean section
c) Pelvic ultrasound
d) Administer steroids
e) Recommend bed rest

Correct!!! An ultrasound should be performed to check for abnormal placentation. Placenta


previa must be ruled out before proceeding with vaginal examination because of the risk of injury
to the placenta and catastrophic bleeding. Bleeding from placenta previa often is without warning
or pain. Cesarean section is not indicated and there is no reason to place her on bed rest. It is not
necessary to administer steroids at this point unless it looks like a delivery is imminent.

c) Pelvic ultrasound would be awarded 10 points

Prepared using: Test Pilot (6.1b43)

A 38-year-old G5P4 woman, with a history of four Cesarean sections, is at 36-weeks gestation
with a singleton pregnancy. She presents to Labor and Delivery with complaints of vaginal
bleeding for the last hour. Prenatal care has been unremarkable except for a second trimester
ultrasound discovering an anterior placenta, which partially covers the cervical os. Follow up
ultrasound exams have confirmed these findings. The patient denies uterine contractions and
abdominal pain. She feels the baby moving. Her blood pressure is 110/60, pulse 110, and she is
afebrile. Her abdomen and uterus are nontender and soft. Fetal heart tones have a baseline of
140 and are reassuring. This patient is at greatest risk for which of the following complications?

a) Vasa previa
b) Placenta accreta
c) Placental abruption
d) Uterine dehiscence
e) Preterm labor

An 18-year-old G1 woman at 32-weeks gestation presents with severe abdominal pain and a
small amount of bleeding. She has received routine prenatal care, smokes one pack of cigarettes
per day and admits to using crack cocaine. On exam, her blood pressure is 140/80, pulse 100
and she is afebrile. Her uterus is tense and very tender. Pelvic ultrasound reveals a fundal
placenta, cephalic presentation of the fetus and no other abnormalities. Cervical examination
reveals blood coming through the os and is one centimeter dilated. Fetal heart tones have a
baseline of 160s, with poor long-term variability and decelerations. Which of the following is the
most likely diagnosis?

a) Placenta previa
b) Premature rupture of the membranes
c) Spontaneous labor
d) Placental abruption
e) Chorioamnionitis

A 21-year-old G1 at 36 weeks of pregnancy presents with sudden onset of abdominal pain and
bleeding. She smokes a pack of cigarettes a day, but otherwise her pregnancy has been
uncomplicated. She takes no medications other than prenatal vitamins. Her blood pressure is
150/90, pulse 90 and she is afebrile. Her uterus is tense and very tender. Pelvic ultrasound
shows the placenta to be posterior and fundal, with a cephalic presentation of the fetus. Cervical
examination reveals no lesions, blood coming through the os and is one centimeter dilated. Fetal
heart tones have a baseline of 150, with poor variability and late decelerations. Tocometer
reveals contractions every 30-45 seconds. Which of the following is the most appropriate next
step in treatment?

a) Amniotomy
b) Cesarean delivery
c) Induction of labor
d) Tocolysis
e) A double set-up examination

A 32-year-old G3P2 woman presents at 40 1/7 weeks gestation because of regular uterine
contractions every 5 minutes for the last 2 hours. Her prenatal course was unremarkable. She
states the baby is moving, but she has had a bright red, bloody discharge for the last 30 minutes.
She does not think she has ruptured her membranes. Her blood pressure is 120/70, pulse 80 and
she is afebrile. Her abdomen is soft and she has regular contractions of moderate intensity. Fetal
heart tones have a baseline of 130 and are reactive with no decelerations. Pelvic ultrasound
reveals a fundal placenta and cephalic presentation of the fetus. Cervical examination reveals a
friable cervix that bleeds easily and is 5 centimeters dilated and completely effaced. Membranes
are confirmed to be intact. Which of the following is the most likely source of bleeding?

a) Placental abruption
b) Placenta previa
c) Bloody show
d) Cervical cancer
e) Cervicitis

A 45-year-old G4P3 woman presents with vaginal bleeding. Last week, she performed a home
pregnancy test that was positive. She thinks her last menstrual period was four months ago. The
last time she saw her doctor was 8 years ago, with the birth of her last child. She has no serious
medical problems, has smoked a pack of cigarettes a day since the age of 20, occasionally has a
beer and does not exercise. Her vitals are normal. Abdominal examination reveals a soft
abdomen and the fundus palpable just below the umbilicus. Pelvic ultrasound reveals a fundal
placenta and a fetus measuring 18 weeks with normal cardiac activity. Vaginal examination
reveals a 3-centimeter lesion arising off the posterior lip of the cervix. It easily bleeds with
palpation and is hard in consistency. Which of the following is the most likely cause of the
bleeding?

a) Trauma
b) Cervicitis
c) Threatened abortion
d) Cervical cancer
e) Nabothian cyst

d) Cervical cancer would be awarded 10 points

Use button to the right. ->


Do NOT use browser Forward or Back Buttons!

A 15-year-old G1P0 woman at 40 weeks gestation presents to Labor and Delivery with
contractions. At 10:00 am, her cervical exam shows that she is 2 centimeters dilated, 70%
effaced and the vertex is at 0 station. Clinical pelvimetry reveals an adequate pelvis and
membranes are intact. The fetus is in a cephalic presentation, with no concern about
macrosomia. Contractions are occurring every 3 to 4 minutes, based on the external monitor.
Fetal surveillance is reassuring and the patient is stable. Her labor slowly progresses and, at 1:00
pm, the patient has spontaneous rupture of membranes. Fetal surveillance remains reassuring.
Her cervical exam is 4 centimeters dilated, 100% effaced, and 0 station. At 4:00 pm, the patient’s
cervical exam is unchanged (4/100/0). Contractions are occurring every 5 to 6 minutes. Which of
the following is the most appropriate next step in the management of this patient?

a) Consent the patient for a Cesarean section secondary to failure to progress


b) Continue fetal surveillance and reexamine the patient in two hours
c) Begin oxytocin augmentation
d) Perform a contraction stress test
e) Have the patient ambulate

Incorrect!!! The patient has an arrest of dilatation in the active phase of labor. She is only
having contractions every 5 to 6 minutes, so it is reasonable to start Pitocin to increase the
frequency and strength of this patient’s contractions. If the patient does not have cervical change
once she is having more frequent contractions on Pitocin, it would be reasonable to place an
IUPC (intrauterine pressure catheter) to assess the strength of the contractions. It is not yet
necessary to perform a Cesarean section. Further observation and having the patient ambulate
do not facilitate delivery. An amnioinfusion is not indicated in this situation.

c) Begin oxytocin augmentation would be awarded 10 points

Prepared using: Test Pilot (6.1b43)

A 22-year-old G1P0 woman at 39-weeks gestation presents to Labor and Delivery in active
labor. She has had an uneventful pregnancy. You suspect a breech presentation and ultrasound
confirms your exam. Which of the following is associated with breech presentation?

a) Gestational diabetes
b) Uterine fibroids
c) Oligohydramnios
d) Macrosomia
e) Mild preeclampsia

Incorrect!!! Prematurity, multiple pregnancy, genetic disorders, polyhydramnios,


hydrocephaly, anencephaly, placenta previa, uterine anomalies and uterine fibroids are all
associated with breech presentation.

b) Uterine fibroids would be awarded 10 points

A 30-year-old G2P1 woman at 38 weeks gestation presents to Labor and Delivery with
contractions every 2 to 3 minutes. Her membranes are intact. Her cervical exam is 5 centimeters
dilated, 100% effaced, and -1 station. The fetal heart rate is reassuring. Two hours later, she
progresses to 7 cm dilated and 0 station. Two hours after that, her exam is unchanged (7/100/0.)
Fetal heart tones are reassuring. Which of the following is the most appropriate next step in the
management of this patient?

a) Allow her to ambulate and return when she is ready to push


b) Perform a contraction stress test
c) Perform an amniotomy
d) Perform a Cesarean Section
e) Place an internal fetal scalp electrode

Incorrect!!! This patient has secondary arrest of dilation, as she has not had any further
cervical change in the active phase for over 2 hours. A multiparous woman should have a rate of
cervical dilation of at least 1.5 cm/hour in the active phase. Amniotomy is often recommended in
this situation. After it is performed, if the patient is still not in an adequate contraction pattern,
augmentation with Pitocin can be attempted after careful evaluation. Although the patient requires
close following and requires active care, it is too early to move to a Cesarean section. An internal
scalp electrode is not necessary, since the fetal heart monitoring is reassuring.

A 76-year-old G3P3 woman presents to your office with worsening urinary incontinence for the
past three months. She reports increase in urinary frequency, urgency and nocturia. Her exam
shows mild cystocele and rectocele. A urine culture is negative. A post-void residual is 400 cc.
Which of the following is the most likely diagnosis in this patient?

a) Genuine stress incontinence


b) Detrusor instability
c) Overflow incontinence
d) Functional incontinence
e) Mixed incontinence

Use button to the right. ->


Do NOT use browser Forward or Back Buttons!

A 76-year-old G3P3 presents to the office with worsening stress urinary incontinence for the
last 3 months. She reports an increase in urinary frequency, urgency and nocturia. On exam, she
has a moderate size cystocele and rectocele. A urine culture is negative. A post-void residual is
50 cc. A cystometrogram shows two bladder contractions while filling. Which of the following is
the most likely diagnosis in this patient?

a) Genuine stress incontinence


b) Urge incontinence
c) Overflow incontinence
d) Functional incontinence
e) Continuous incontinence

Incorrect!!! Detrusor overactivity incontinence is also known as urge incontinence. Detrusor


instability is due to the overactivity of the bladder muscle. Though the testing may be simple
(using a Foley catheter and attached large syringe without the plunger, filling with 50-60 cc of
water at a time) or complex (using computers and electronic catheters,) the uninhibited
contraction of the bladder with filling makes the diagnosis.

b) Urge incontinence would be awarded 10 points

Prepared using: Test Pilot (6.1b43)

7. A 57-year-old G2P2 woman presents to the office with a six-month history of leaking urine,
urgency, and nocturia. She describes the amount of urine loss as large and lasting for several
seconds. The urine loss occurs when she is standing or sitting and is not associated with any
specific activity. What is the most likely cause of this patient’s symptoms?

a) Stress incontinence
b) Overflow incontinence
c) Urge incontinence
d) Mixed incontinence
e) Vesicovaginal fistula

c
A 90-year-old G7P7 woman presents with severe vaginal prolapse. The entire apex, anterior
and posterior wall are prolapsed beyond the introitus. She cannot urinate without reduction of the
prolapse. Hydronephrosis was noted on ultrasound of the kidneys and it is thought to be related
to the prolapse. She has a long-standing history of diabetes and cardiac disease. She is not a
candidate for general or regional anesthesia. She has failed a trial of pessaries. Which of the
following is the next best step in the management of this patient?

a) Do nothing and observe


b) Anterior and posterior repair
c) Colpocleisis
d) Sacrospinous fixation
e) Sacrocolpopexy

Incorrect!!! Because of the hydronephrosis due to obstruction, intervention is required.


Anterior and posterior repairs provide no apical support of the vagina. She will be at high risk of
recurrent prolapse. The sacrospinous fixation (cuff to sacrospinous-coccygeus complex) or
sacrocolpopexy (cuff to sacral promontory using interposed mesh) require regional or general
anesthesia. Colpocleisis is a procedure where the vagina is surgically obliterated and can be
performed under local anesthesia. Recurrence is minimal.

c) Colpocleisis would be awarded 10 points

A 63-year-old G0 woman comes to the office for a health maintenance exam. She is healthy and
not taking any medications. She has no history of abnormal Pap smears or sexually transmitted
diseases. She has a history of endometriosis and infertility in the past. She has been
postmenopausal for 10 years and is not on hormone replacement therapy. She is 5 feet 4 inches
tall and weighs 130 pounds. On pelvic examination, the patient has a palpable left adnexal mass.
An ultrasound was obtained, which showed a 5 cm complex left ovarian cyst. What is the most
appropriate next step in the management of this patient?

a) CAT scan of the abdomen and pelvis


b) MRI of the pelvis
c) Exploratory surgery
d) Repeat ultrasound in 3 months
e) Observation

A 48-year-old G0 woman comes to the office for a health maintenance exam. She is healthy and
not taking any medications. She has no history of abnormal Pap smears or sexually transmitted
infections. Her menstrual cycles are normal and her last cycle was three weeks ago. Her mother
was diagnosed with endometriosis and had a hysterectomy and removal of the ovaries at age 38.
She is 5 feet 4 inches tall and weighs 130 pounds. On pelvic examination, the patient had a
palpable left adnexal mass. An ultrasound was obtained, which showed a 4 cm complex left
ovarian cyst and a 2 cm simple cyst on the right ovary. What is the most likely diagnosis in this
patient?

a) An endometrioma
b) A hemorrhagic cyst
c) Ovarian carcinoma
d) A mature teratoma
e) Polycystic ovaries

A 29-year-old G0 woman presents due to the inability to conceive for the last two years. She has
a known history of endometriosis, which was diagnosed by laparoscopy 3 years ago. She has
pelvic pain, which is controlled with non-steroidal anti-inflammatory drugs. Her cycles are regular
every 28 days. She is otherwise in good health and has been married for 5 years. Her husband
had a semen analysis, which was normal. She had a hysterosalpingogram, which showed patent
tubes bilaterally. She is getting frustrated that she has not yet gotten pregnant and asks to
proceed with fertility treatments. What is the most appropriate next step in the management of
this patient?

a) Perform a laparoscopy
b) Administer a GnRH agonist
c) Ovarian stimulation with Clomiphene Citrate
d) Proceed with in vitro fertilization
e) Intrauterine insemination

A 48 year-old woman presents with complaints of a white, watery nipple discharge for 4 months.
She has been told in the past she had fibrocystic breast changes, but otherwise has no significant
medical problems or surgical history. The white nipple discharge is noted on manual expression,
but the exam is otherwise normal. Her serum prolactin level was 45 ng/ml (normal below 40
ng/ml). What is the most appropriate next step in the management of this patient?

a) Obtain a brain MRI


b) Obtain a β- HCG
c) Begin Bromocriptine
d) Obtain a fasting prolactin level
e) Order a ductogram

A 42 year-old G3P3 patient comes to the office after noticing a breast mass while performing a
breast self-exam. She is in good health and has normal menstrual cycles. Family history is
significant for multiple first and second degree relatives having breast cancer. Physical exam
reveals a 2-centimeter dominant breast mass. The remainder of the exam is normal. A
mammogram obtained today shows no abnormalities. What is the most appropriate next step in
the management of this patient?

a) Reassurance and observation


b) Obtain genetic testing
c) MRI of the breast
d) Fine needle aspiration
e) Repeat mammogram in 2 months

A 42 year-old G3P3 patient comes to the office after noticing a breast mass while performing a
breast self-exam. She is in good health and has normal menstrual cycles. Physical exam is
significant for a 2 cm dominant breast mass. The remainder of the exam is normal. A
mammogram obtained today shows no abnormalities. A fine needle aspiration was negative, and
the mass persisted. What is the most appropriate next step in the management of this patient?

a) Reassurance and observation


b) Obtain CAT scan of the chest
c) Breast ultrasound
d) Perform an excisional biopsy
e) Repeat mammogram in 2 months

Incorrect!!! A specimen obtained on fine-needle aspiration is examined both histologically


and cytologically. An excisional biopsy should be performed when the results are negative, due to
the possibility of a false-negative result. It can, however, prevent the need for other diagnostic
testing and is the appropriate next step. Breast ultrasound can be used to distinguish between a
cyst and a solid mass. Fine needle aspiration under ultrasound guidance can help distinguish a
fibroadenoma from a cyst and exclude cancer in certain situations. A normal mammogram does
not rule out breast cancer and there is no need to repeat it in 2 months. There are no indications
for obtaining a CAT scan of the chest in the initial diagnosis of this patient.

A 24-year-old G0 woman presents with a one-year history of introital and deep thrust
dyspareunia. She also has a 2-year history of severe dysmenorrhea, despite the use of oral
contraceptives. She underwent a diagnostic laparoscopy 6 months ago that showed minimal
endometriosis with small implants in the posterior cul de sac only, which were ablated with a CO2
laser. On further questioning, she reports significant urinary frequency, urgency and nocturia. A
recent urine culture was negative. The most likely additional diagnosis for this patient is:

a) Acute cystitis
b) Interstitial cystitis
c) Acute urethral syndrome
d) Acute urethritis
e) Salpingitis

A 22-year-old G0 woman graduate student, presents with worsening pelvic pain. She previously
underwent a laparoscopic ablation of endometriosis followed by continuous oral-contraceptive
pills. She had short-term relief from this approach, but now has failed this treatment and is
seeking additional medical management. Which of the following mechanisms best explains how a
gonadotropin releasing hormone (GnRH) agonist would help alleviate her symptoms?

a) "Down-regulation” of the hypothalamic-pituitary gland production and release of LH and FSH


leading to reduction in estradiol levels
b) “Up-regulation” of the hypothalamic-pituitary gland production and release of LH and FSH
leading to elevation in estradiol levels
c) Suppression of both LH and FSH mid-cycle surges, resulting in the loss of ovarian
production of estrogen
d) Induction of a “pseudopregnancy” state, causing a decidual reaction in the functioning
endometrial tissue
e) Competitive inhibitor for estrogen receptors

A 62-year-old G5P5 grandmother, menopausal for 12 years, presents with a 7-month history of
pelvic pain and pressure, as well as abdominal distention and bloating. She experiences
occasional constipation, but no melena or hematochezia. She also has mild to moderate urinary
frequency without dysuria, hematuria or flank pain. She describes no exacerbating or alleviating
factors. Her medical history is significant for hypertension and obesity. She has never been on
hormone therapy. She notes one episode of light vaginal bleeding several months ago. Her family
history is significant for postmenopausal ovarian cancer in her mother and maternal aunt, but is
otherwise negative for breast, endometrial or colon carcinoma. Pelvic examination is remarkable
for vaginal atrophy, cervical stenosis and difficult uterine and adnexal assessment due to her
body habitus. What is the most appropriate next step in the management of this patient?

a) Performing a transvaginal ultrasound


b) Diagnostic laparoscopy
c) Computed Axial Tomography (CT) scan of the abdomen and pelvis
d) Colonoscopy
e) Hysteroscopy

Incorrect!!! Given the patient’s age, nonspecific abdomino-pelvic symptoms, recent


postmenopausal bleeding episode and family history of ovarian cancer, a transvaginal ultrasound
is the next best step as it is more sensitive than CT for evaluation of the uterus and adnexa.
Endometrial thickness of 4 mm or less on transvaginal ultrasound in postmenopausal women is
rarely associated with endometrial malignancy. Colonoscopy is useful for colorectal cancer
screening, as well as evaluation of the patient’s gastrointestinal symptoms, but would not provide
information regarding pelvic anatomy. Diagnostic laparoscopy would be a more invasive
procedure that could be performed as indicated, after these other diagnostic studies.
Hysteroscopy might be useful based on the ultrasound results, since it might be difficult to
perform an endometrial biopsy in the office.

a) Performing a transvaginal ultrasound would be awarded 10 points

Use button to the right. ->


Do NOT use browser Forward or Back Buttons!

A 33-year-old G2P2 woman reports a 2-year history of severe dysmenorrhea, menorrhagia and
pelvic pain immediately following the forceps-assisted delivery of her last child, which was
complicated by postpartum hemorrhage and right vaginal sidewall laceration. She describes her
pelvic pain as primarily in the right lower quadrant, radiating into the vagina. Her pain worsens
throughout the day with standing and is associated with pelvic pressure and fullness. Her pelvic
examination reveals a mildly enlarged uterus with marked tenderness to palpation of the right
adnexa, and no other significant findings. A vaginal ultrasound with color-flow Doppler reveals
multiple dilated vessels traversing the right broad ligament to the lower uterus and cervix. The
uterus shows no fibroids or other significant changes. Endometrial thickness appears normal.
Which of the following is the most likely diagnosis in this patient?

a) Endometriosis
b) Endometritis
c) Adenomyosis
d) Pelvic congestion
e) Pelvic floor relaxation

Correct!!! The pelvic congestion syndrome is a cause of chronic pelvic pain occurring in the
setting of pelvic varicosities. The unique characteristics of the pelvic veins make them vulnerable
to chronic dilatation with stasis leading to vascular congestion. These veins are thin walled and
unsupported, with relatively weak attachments between the supporting connective tissue. The
cause of pelvic vein congestion is unknown. Hormonal factors contribute to vasodilatation when
pelvic veins are exposed to high concentration of Estradiol, which inhibits reflex vasoconstriction
of vessels, induces uterine enlargement with selective dilatation of ovarian and uterine veins. This
pain may be of variable intensity and duration, is worse premenstrually and during pregnancy,
and is aggravated by standing, fatigue and coitus. The pain is often described as a pelvic
“fullness” or “heaviness,” which may extend to the vulvar area and legs. Associated symptoms
include vaginal discharge, backache and urinary frequency. Menstrual cycle defects and
dysmenorrhea are common. No signs of pelvic floor relaxation were noted on exam.

d) Pelvic congestion would be awarded 10 points

Prepared using: Test Pilot (6.1b43)

A 45-year-old G2P2 woman underwent an abdominal hysterectomy for a large fibroid uterus via a
low transverse skin incision. Her postoperative course was significant for new onset right lower
quadrant pain and numbness, radiating into the right inguinal area and medial thigh. Her pain was
exacerbated by adduction of her right thigh. On abdominal examination, there is a well-healed low
transverse incision. Her pain is reproduced with adduction of the right thigh. There is decreased
sensation to light touch and pinprick over the right inguinal area and right medial thigh. Patellar
reflexes are 2+ and symmetric. Entrapment of which of the following nerves is the most likely
cause of her pain?

a) Obturator nerve
b) Ilioinguinal nerve
c) Lateral femoral cutaneous nerve
d) Femoral nerve
e) Iliohypogastric

A 26 year-old G0 was found to have a low-grade squamous intraepithelial lesion (LGSIL) on


routine Pap smear. She underwent a colposcopy with cervical biopsy. Her colposcopy was
adequate and biopsy results showed CIN-I. There was no endocervical glandular involvement.
Endocervical curettage showed benign cells. Which of the following is the most appropriate
treatment for this patient?

a) Cold knife conization


b) Loop Electrosurgical Excision Procedure (LEEP)
c) Cryotherapy
d) Follow up Pap smear in 6 months
e) Complete removal of the lesion

A 38 year-old G1P1 woman comes to the office for an annual exam. She has noticed some
urinary frequency over the past month. She has no dysuria, hematuria, urgency or incontinence.
She has normal cycles, no history of abnormal Pap smears or sexually transmitted infections and
is sexually active, with 1 partner. She smokes a quarter of a pack of cigarettes daily, and drinks
one glass of wine per day. Her mother had breast cancer at age 30. Her general examination is
normal. On pelvic exam, she has normal external genitalia; vagina and cervix are without lesions.
Her uterus is normal size, anteverted and nontender. Her left adnexa is normal, right adnexa has
a mobile, slightly tender 4 cm mass. Laboratory results show a normal urinalysis, a negative urine
pregnancy test and a normal Pap smear. What is the most appropriate next step in the
management of this patient?

a) Perform a transvaginal ultrasound


b) Perform a diagnostic laparoscopy
c) Recommend a CT-guided drainage of the mass
d) Order a KUB plain film
e) Perform an exploratory laparotomy

A 7 year-old is undergoing evaluation for vaginal bleeding. On physical examination, she has
Tanner III stage breasts, tall stature and an otherwise normal examination. An MRI of the brain
and a pelvic ultrasound are normal. LH and FSH levels are in the pre-pubertal levels and she has
normal DHEAS and androgen levels. What is the most likely diagnosis in this patient?

a) Pituitary adenoma
b) Congenital adrenal hyperplasia
c) True precocious puberty
d) Testicular feminization
e) Ovarian neoplasm

Use button to the right. ->


Do NOT use browser Forward or Back Buttons!

A 17 year-old is brought to the physician because she has never had a menstrual cycle. She
has normal breast and pubic hair development. Physical examination reveals a small vaginal
opening with a blind pouch. Pelvic ultrasound reveals normal ovaries, but absence of uterus and
cervix. Which of the following is the most appropriate next study in this patient?

a) Renal ultrasound
b) FSH and LH determination
c) Karyotype
d) Cortisol level
e) Testosterone level

Incorrect!!! Renal anomalies occur in 25-35% of females with Mullerian agenesis. The uterus
and cervix are absent, but the ovaries function normally and, therefore, secondary sexual
characteristics are present. You would expect the karyotype in this patient to be 46,XX.

a) Renal ultrasound would be awarded 10 points

Prepared using: Test Pilot (6.1b43)

A 13 year-old girl is brought to the physician for increasingly severe abdominal pain. The pain
is now a constant low discomfort, but every month she has a week when it is more severe. She
has Tanner stage II breast and pubic hair development. On genital examination, there is a bluish
mass pushing the labia open. What is the most likely cause of this patient’s abdominal pain?

a) Turner’s syndrome
b) Transverse vaginal septum
c) Mullerian agenesis
d) Imperforate hymen
e) Synechiae of the uterine cavity

Incorrect!!! Lower genital tract malformations occur in 1 in 10,000 females and are most
commonly an imperforate hymen where the genital plate canalization is incomplete. The
menstrual blood will collect in the vagina and uterus causing pain. Treatment involves surgical
correction. When a transverse vaginal septum is present, a normal vaginal opening with a short
blind vagina and pelvic mass may be located above the level of the obstruction found on exam.
Asherman’s syndrome is associated with secondary amenorrhea resulting from intrauterine
scarring/synechiae.

A 24-year-old woman comes into the office because she has not had her menses for 6 months.
She is in good health and not taking any medications. She is not sexually active. She does well in
graduate school, despite her demanding new program. Her height is 5’ 6” and her weight is 104
pounds. Her blood pressure is 140/80, with other vital signs being stable. Her physical
examination, including a pelvic examination, is completely normal. What is the most likely reason
for her amenorrhea?

a) Ovarian dysfunction
b) Undiagnosed diabetes mellitus
c) Obstruction of the genital outflow tract at the level of the endocervical canal
d) Hypothalamic-Pituitary Dysfunction
e) Pregnancy

Correct!!! Anorexia nervosa or significant weight loss may cause hypothalamic-pituitary


dysfunction that can result in amenorrhea. A lack of the normal pulsatile secretion of
gonadotropin releasing hormone (GnRH) leads to a decreased stimulation of the pituitary gland to
produce follicle stimulating hormone (FSH) and luteinizing hormone (LH.) This leads to
anovulation and amenorrhea. Diabetes mellitus does not primarily result in amenorrhea. While
ovarian dysfunction/failure, obstruction of the genital outflow tract and pregnancy cause
amenorrhea, they are unlikely in this case.

A 23-year-old woman presents to the office because she has not had any menses for 4 months.
She has a long history of irregular menstrual cycles since menarche at age 14. She is otherwise
in good health and is not taking any medications. She is thin and has chronic anxiety. Her beta-
HCG is < 5, and her prolactin and TSH levels are normal. You suspect hypothalamic-pituitary
dysfunction as the cause of her amenorrhea. What would be the next best diagnostic test to
order?

a) Estrogen level
b) Transvaginal pelvic ultrasound
c) Gonadotropin releasing hormone level
d) Follicle stimulating hormone and luteinizing hormone levels
e) Progesterone level

Use button to the right. ->


Do NOT use browser Forward or Back Buttons!

A 22-year-old G0 female student comes to the office because she has not had any menses
since discontinuing her oral contraceptive pills 5 months ago. She has been on the pill for the last
6 years and had normal menses every 28 days while taking them. She is in good health and not
taking any medications. She is 5’4” tall and weighs 130 pounds. Her examination, including a
pelvic exam, is normal. Which of the following history elements would be most useful in
determining the cause of amenorrhea in this patient?

a) Age at first intercourse


b) History of sexually transmitted diseases
c) Parity
d) History of oligo-ovulatory cycles
e) Recent history of weight loss

Incorrect!!! Since most women resume normal menstrual cycles after discontinuing oral
contraceptive pills (OCPs,) they are not usually considered the cause of the amenorrhea. A
history of irregular cycles prior to pill use may increase the risk of amenorrhea upon
discontinuation. This is sometimes referred to as “post pill amenorrhea.” A complete work-up
should be performed to properly find the cause. Although the other history elements are all
important components of a complete gynecological history, they are not helpful to find the etiology
of amenorrhea in this patient. Significant weight loss might cause amenorrhea; however, this
patient still has normal body mass index, which makes it unlikely cause of amenorrhea.

d) History of oligo-ovulatory cycles would be awarded 10 points


Test Pilot (6.1b43) is Copyright ©2006, The McGraw Hill Companies, All Rights Reserved.

An 18 year-old G0 woman presents with a 1-year history of hirsutism and acne. She had
menarche at age 14 and her menses have been irregular every 26-60 days. Her sister has a
similar pattern of hair growth. The patient is 5’4” tall and weighs 180 pounds. On exam, a few
terminal hairs were identified on her chin and upper lip. TSH, prolactin, total testosterone, and
DHEAS levels are normal. Which of the following is the most appropriate next test to evaluate this
patient’s condition?

a) Estradiol levels
b) Serum cortisol levels
c) Urinary cortisol levels
d) Random blood glucose
e) 17-hydroxyprogesterone

Incorrect!!! Checking 17-hydroxyprogesterone would rule out late onset 21-hydroxylase


deficiency. Normal TSH, Prolactin, total testosterone and DHEAS levels rule out pituitary or
adrenal tumors. The patient could have polycystic ovarian syndrome; however, normal serum
testosterone levels make it less likely.

A 34 year-old G2P2 presents with concerns of hormonal changes. She is worried about facial
hair growth, worsening acne, and deepening of her voice. She also realized that she has missed
her period for 2 months, and has been sexually active and had tubal ligation for contraception. On
examination, she is moderately obese and noted to have severe acne, upper lip and chin terminal
hair. Her abdomen is obese with moderate hair growth. Pelvic examination is most notable for an
enlarged clitoris, and pelvic exam reveals an enlarged right sided adnexal mass. Which of the
following is the most likely diagnosis in this patient?

a) Sertoli-Leydig cell tumors


b) Granulosa cell tumor
c) Benign cystic teratoma
d) Choriocarcinoma
e) Cystadenoma

Correct!!! The most likely diagnosis in this patient is a testosterone-secreting ovarian tumor.
Sertoli-Leydig cell tumors are commonly diagnosed in women between the ages of 20-40, and
are most often unilateral. Rapid onset of hirsutism and virilizing signs are hallmarks of this
disease, and include many of the findings in this patient including acne, hirsutism, amenorrhea,
clitoral hypertrophy, and deepening of the voice. Abnormal laboratory findings include
suppression of FSH and LH, marked elevation of testosterone, and presence of an ovarian mass.
The constellation of findings is most consistent with a testosterone-secreting tumor, and a pelvic
ultrasound will confirm the presence of an ovarian mass. The other tumors do not cause
virilization. Granulosa cell tumors are estrogen secreting and choriocarcinoma is notable for
elevated BHCG.

A 26 year-old woman comes to the office due to irregular menses since menarche, worsening for
the last 6 months. The patient has noted increasing hair growth on her chin and most recently
hair growth on her chest, requiring that she shave periodically. No one in her family has hirsutism.
On exam, you also notice acne on her chin, acanthosis nigricans and temporal balding. Her
serum testosterone is elevated. You suspect hyperthecosis. Which of the following might also be
associated with this condition?

a) Hyperthyroidism
b) Hyperprolactinemia
c) Atrophic changes of external genitalia
d) Deepening of the voice
e) Hyperparathyroidism

A 34 year-old G2P2 woman presents with inter-menstrual bleeding for one year. The bleeding
typically occurs 2 weeks after her menses and last 2-3 days. The symptoms began 1 year ago
and the bleeding has not changed recently. She is currently taking oral contraceptives. On pelvic
examination, the cervix appears normal and the uterus is normal in size and shape. Her urine
pregnancy test is negative; an endometrial biopsy is negative for neoplasia. Which of the
following tests or procedures would be indicated for further work-up?

a) Prolactin level
b) Progesterone level
c) Hysterosalpingogram (HSG)
d) Pelvic ultrasound
e) Colposcopy

A 45 year-old G2P2 woman comes to the office because of heavy and irregular menstrual
periods. The heavy periods started three years ago and have gradually worsened in amount of
flow over time. The periods are interfering with her daily activities. The patient has had two
spontaneous vaginal deliveries, followed by a tubal ligation 3 years ago. On pelvic examination,
the cervix appears normal and the uterus is normal in size without adnexal masses or
tenderness. A urine pregnancy test is negative. TSH and prolactin levels are normal. Hemoglobin
is 12.5 mg/dl. On pelvic sonography, she has a normal size uterus and a 2 cm simple cyst on the
right ovary. Endometrial biopsy is consistent with a secretory endometrium; no neoplasia is found.
What is the most likely diagnosis in this patient?

a) Polycystic ovarian syndrome


b) Mid-cycle bleeding
c) Dysfunctional uterine bleeding
d) Benign cystic teratoma
e) Ovarian cancer

Vous aimerez peut-être aussi